Übergangs-Marathon Mathematik - Seite 5

Neue Frage »

HAL 9000 Auf diesen Beitrag antworten »

Ein Versuch, den Thread zu reaktivieren:

Zitat:
Aufgabe 57

Für jede ganze Zahl sei die Folge gegeben durch

.

Man bestimme alle Werte von , so dass beschränkt ist.


Die Funktionalgleichung ist auch nicht übel:

Zitat:
Aufgabe 58

Man bestimme alle Funktionen , so dass für alle die Gleichung



gilt.

Beide noch gut machbar, die 58 ist allerdings schon ein Stückchen schwerer.



P.S.: Ähnlichkeiten mit Aufgaben eines kürzlich stattgefundenen (abgeschlossenen!) Wettbewerbs sind durchaus beabsichtigt. Big Laugh
Clearly_wrong Auf diesen Beitrag antworten »

Also mir hat die Aufgabe sehr gut gefallen, die verstrichene Zeit darf nicht als Desinteresse gewertet werden, ich wollte nur ein bisschen abwarten, ob sich noch jemand anderes daran versuche möchte.
Die Aufgabe 58 habe ich gerade erst gesehen, die schaue ich mir dann auch an.

Zitat:

Lösung 57

Die Folge bleibt genau dann beschränkt, wenn .

Beweis: Falls , so ist die Folge beschränkt durch die erste durch teilbare Quadratzahl oberhalb von . Das ist anschaulich sofort klar, hier dennoch ein kleines Argument: Jedes Folgenglied der Folge ist in diesem Fall durch teilbar, da die Quadratwurzel einer durch teilbaren Zahl ebenfalls durch teilbar ist. Wenn es ein gäbe, dann wähle ein kleinstes solches. Dann muss notwendigerweise gelten, also , Widerspruch.

Falls , so wächst die Folge unbeschränkt, da Quadratzahlen niemals Rest bei Division durch lassen.

Es gelte nun . Angenommen, die Folge bliebe beschränkt. Dann muss jedes der Folgenglieder Rest lassen, denn Rest ist durch die Operationen der Folge nicht erreichbar und Rest macht die Folge sofort unbeschränkt. Wähle eine kleinste Zahl , die unendlich oft als Folgenglied angenommen wird. Da wieder als Folgenglied erreicht wird, muss auch ein Folgenglied sein. Das heißt aber, dass zwischen und keine Quadratzahl liegt, die Rest bei Division durch lässt, andernfalls hätten wir einen Widerspruch zur Minimalität von . Dann muss aber die Quadratzahl unterhalb von liegen, also gelten. Daraus folgt , ein Widerspruch.
HAL 9000 Auf diesen Beitrag antworten »

Ja, so einfach bekommt man 7 Punkte bei der IMO. Freude
Leopold Auf diesen Beitrag antworten »

Ich habe einmal nach einfachen Funktionen gesucht, die der Funktionalgleichung von Aufgabe 58 genügen. Denkt man an ganzrationale Funktionen höchstens vom Grad 1, also

,

und geht man damit in die Funktionalgleichung, so führt ein Koeffizientenvergleich auf die Kombinationen oder oder . Und tatsächlich erfüllen die Funktionen mit



die Funktionalgleichung. Was aber die Eindeutigkeit angeht, tappe ich noch im Dunkeln.
Clearly_wrong Auf diesen Beitrag antworten »

Zitat:

Lösung 58
Die einzigen Lösungen der Funktionalgleichung sind die Nullfunktion, und . Dass diese Funktionen die Gleichung erfüllen ist einfaches Nachrechnen.

Sei nun eine Lösung der Funktionalgleichung. Da mit auch eine Lösung ist, wie man leicht nachrechnet, sei ohne Einschränkung . Falls , so ist für jedes , also ist die Nullfunktion. Sei also im Folgenden .

Zunächst bemerken wir, dass, wie von HAL im Diskussionsthread vorgeschlagen, für jedes gilt, dass , also hat eine Nullstelle. Angenommen es gibt eine Nullstelle von ungleich . Dann gilt , also ist wie oben gesehen die Nullfunktion. Das heißt ist die einzige Nullstelle von .

Nun gilt , also ist und damit . Wegen ist .

Weiter gilt für jedes .

Wir zeigen Injektivität von . Seien mit .

Betrachte die quadratische Gleichung . Für groß genug hat diese Gleichung zwei reelle Lösungen .
Nach dem Satz von Vieta erfüllen diese und . Setzen wir die Lösungen also in die Funktionalgleichung ein, folgt
, also . Es folgt . O.B.d.A. sei . Es folgt und damit , also .

Weiter gilt , also wegen Injektivität .

Schlussendlich haben wir und wegen Injektivität . Es folgt .

RavenOnJ Auf diesen Beitrag antworten »

Als nächstes wäre dann die Aufgabe 3 mit dem Jäger und unsichtbaren Hasen dran. Vielleicht hat da ja auch jemand eine Idee. Die desaströse Punkteausbeute bei dieser Aufgabe spricht für sich. (2 x 7, 1 x 5, 1 x 4, 3 x 1 der Rest 0)
 
 
Leopold Auf diesen Beitrag antworten »

Zitat:
Original von RavenOnJ
Als nächstes wäre dann die Aufgabe 3 mit dem Jäger und unsichtbaren Hasen dran.


Ich weiß nicht, wovon du sprichst. verwirrt

Derweil mal eine Aufgabe, bei der man vielleicht keine geniale Idee braucht, sondern eher eine eingeschlagene Lösungsstragie konzentriert und fehlerfrei zu Ende bringen muß.

Zitat:
Aufgabe 59

Man denke sich zwei kongruente Quadrate übereinander liegend, so daß sie sich vollständig decken. Dann drehe man das aufliegende Quadrat mit 45° um seinen Mittelpunkt und hebe es senkrecht nach oben an. Nun wird jede Ecke eines der Quadrate mit den beiden nächstliegenden Ecken des anderen Quadrates durch eine Strecke verbunden. So entsteht ein Körper mit Quadraten als Boden- und Deckfläche und einer Mantelfläche aus acht gleichschenkligen Dreiecken. Der Abstand der beiden Quadrate sei so gewählt, daß alle Kanten des Körpers gleich lang sind, die Mantelfläche also aus gleichseitigen Dreiecken besteht.
Den beschriebenen Körper nennen wir quadratisches Antiprisma.
Man bestimme das Volumen des quadratischen Antiprismas in Abhängigkeit von der Kantenlänge des Körpers.
HAL 9000 Auf diesen Beitrag antworten »

Zitat:
Original von Leopold
Zitat:
Original von RavenOnJ
Als nächstes wäre dann die Aufgabe 3 mit dem Jäger und unsichtbaren Hasen dran.

Ich weiß nicht, wovon du sprichst. verwirrt

Wenn ich das aufklären darf: https://artofproblemsolving.com/communit..._2017_problem_3
RavenOnJ Auf diesen Beitrag antworten »

@HAL
Danke für den Link.

@ Leopold
Das war die Hammeraufgabe 3 aus der IMO 2017, die fast keiner dort gelöst hatte, noch nicht mal ansatzweise. Das Ergebnis hatte ich ja schon erwähnt.
Leopold Auf diesen Beitrag antworten »

Dann jetzt hier einmal die Lösung nach Huggy. Ich habe seine Zeichnung herüberkopiert und ergänzt.

[attach]45039[/attach]

Die Kantenlänge des Körpers sei 2. Also hat das gleichseitige Dreieck die Höhe . Die Höhe des Körpers ist Kathete in einem rechtwinkligen Dreieck, dessen andere Kathete der Überhang ist und dessen Hypotenuse die Höhe von ist. Also gilt und somit



Das umschließende Prisma hat als Grundfläche ein regelmäßiges Achteck. Man erhält sie, indem man zum blauen Quadrat vier gleichschenklige Dreiecke mit Basis 2 und Höhe , also mit der Grundfläche , dazunimmt:



Das umschließende Prisma hat folglich das Volumen



Davon sind jetzt noch 8 Pyramiden mit Grundfläche und Höhe abzuziehen. Eine von ihnen hat das Volumen



Das Volumen des Antiprismas mit Kantenlänge 2 ist daher



Und für das Volumen mit Kantenlänge hat man das Ergebnis noch mit zu multiplizieren:

Mathema Auf diesen Beitrag antworten »

Beim kurzen Überfliegen ist mir aufgefallen, daß ein Themengebiet hier noch fehlt (ich hoffe ich habe nichts übersehen):

Zitat:
Aufgabe 60:

In einer Urne befinden sich grüne und rote Kugeln, insgesamt sind es Stück (). Es werden gleichzeitig 2 Kugeln aus der Urne gezogen und folgende Ereignisse definiert:
A: Beide Kugeln sind gleichfarbig
B: Beide Kugeln haben eine unterschiedliche Farbe.
Ferner soll gelten . Für welche ist dieses möglich und wie viele grüne und rote Kugeln liegen in diesem Fall in der Urne?
Leopold Auf diesen Beitrag antworten »

Hübsche zahlentheoretische Aufgabe im Wahrscheinlichkeitsgewand mit verblüffender Lösung.
Leopold Auf diesen Beitrag antworten »

Lösung Aufgabe 60

seien die Anzahlen der roten bzw. grünen Kugeln. Dann gilt zunächst



Die Bedingung führt auf



Eliminiert man , so erhält man nach ein paar Umformungen



Also muß eine Quadratzahl sein, etwa mit einer positiven ganzen Zahl . Es folgt







Offenbar fällt positiv ganzzahlig aus. Aus kann man auch noch berechnen. Zusammenfassend hat man für die Lösungen des Problems die Parameterdarstellung



Zitat:
Aufgabe 61

Auf einer Kreislinie werden Punkte unabhängig voneinander zufällig bestimmt (für die Wahl eines Punktes nimmt man Gleichverteilung an).
Mit welcher Wahrscheinlichkeit kann man die Punkte zu einem Sehnen--Eck verbinden, so daß der Mittelpunkt des Kreises im Innern des Sehnen--Ecks liegt?


Für die Aufgabe habe ich mir eine Lösung überlegt, bin aber nicht ganz sicher, ob sie stimmt. Sollte meine Lösung richtig sein, ergibt sich ein relativ übersichtlicher Ausdruck in für die gesuchte Wahrscheinlichkeit, was mich vermuten läßt, daß meine umfangreichen Berechnungen die "knackige Idee dahinter" verbergen. Wer findet eine "elegante" Lösung?
Leopold Auf diesen Beitrag antworten »

Zur Lösung von Aufgabe 61 mit Idee von Mathema, Präzisierung von HAL und Formalisierung von Leopold.

Dann eine weitere Aufgabe.

Zitat:
Aufgabe 62
[attach]45053[/attach]
bis sind der Reihe nach die Seitenmitten eines geschlossenen Polygonzugs. Man bestimme den Polygonzug.
xb Auf diesen Beitrag antworten »

Hier die Lösung (zumindest mal das Bild)
xb Auf diesen Beitrag antworten »

Lösung 62

Man beginnt mit dem 1.Eckpunkt


Zum 2. Eckpunkt kommt man so


Das geht dann immer so weiter
Der "6.Eckpunkt" ist aber wieder der 1. Eckpunkt,weil es ja nur 5 Eckpunkte gibt



Das kann man zusammenfassen







Die Koordinaten der Mittelpunkte muss man möglichst genau messen
xb Auf diesen Beitrag antworten »

Ich hatte diese Aufgabe vor längerer Zeit im Schüler-Marathon gestellt
Sie wurde aber abgelehnt

Deshalb hier ein neuer Versuch

Man hat einen Zahlenstrahl mit den natürlichen Zahlen
Vorne die Null

Bei der folgenden Irrfahrt startet man auf der Null
und kann sich mit jedem Schritt immer nur von einer Zahl auf eine Nachbarzahl bewegen

Ich will jetzt mit 2 Schritten wieder auf der Null sein
Ich gehe also auf die 1 und dann wieder auf die Null
Man hat also genau eine Möglichkeit um mit 2 Schritten wieder auf der Null zu sein

Jetzt will ich in 8 Schritten wieder auf der Null sein
das wäre zB 1-2-1-2-1-0-1-0
Man kann also zwischendurch immer mal wieder auf der Null sein

Frage
Wie viele unterschiedliche Möglichkeiten hat man wenn man mit 8 Schritten wieder auf der Null sein will?[/quote]

Die Lösung lautet


M=Anzahl der Möglichkeiten; s=Anzahl der Schritte

M(8)=14

Gehen von der Null 2 Zahlenstahlen aus dann sieht die Formel so aus

M(8)=70


Aufgabe 63

Jetzt gehen von der Null n Zahlenstrahlen weg (immer noch mit den natürlichen Zahlen)

Wie viele Möglichkeiten hat man jetzt, wenn in 8 Schritten wieder auf der Null sein will?
(Man darf sich nur auf den Zahlenstrahlen bewegen und nicht im Raum dazwischen)
HAL 9000 Auf diesen Beitrag antworten »

Zitat:
Lösung 63

Sei die Anzahl der Pfade, bei Strahlen in Schrítten wieder zur Null zurückzukehren.

Wie von dir angemerkt ist . Außerdem ist die Anzahl der Pfade, auf einem einmal beschrittenen Strahl erstmalig (!) nach Schritten wieder zur Null zurückzukehren. D.h., auf einem solchen Pfad hat man zwischendurch nicht die Möglichkeit, den Strahl zu wechseln - das ist ja erst wieder möglich, wenn man die Null erreicht hat,

Nun haben wir Start sowie die Iterationsgleichung

für ,

die lässt sich so erklären: Am Start hat man die Wahl von Strahlen, die man zuerst beschreitet. Dort verweilt man dann Schritte, bis man erstmalig zur Null zurückkehrt. Von dort hat man dann mögliche Pfade für die restlichen Schritte.

(*) in Verbindung mit dem Rekursionsstart liefert



Und ist ja die von dir gesuchte Zahl, du hattest ja bereits und angegeben.

Ob es für allgemeine auch eine schöne griffige explizite Formel für gibt, bleibt noch zu erforschen. Augenzwinkern
Huggy Auf diesen Beitrag antworten »

Mal etwas Spielerisches:

Zitat:
Aufgabe 64

Zwei Personen A und B vereinbaren folgendes Spiel. Beide schreiben verdeckt eine der Zahlen 1 oder 2 auf. Dann wird aufgedeckt. sei die Summe der beiden Zahlen. Ist gerade, zahlt B Taler an A. Ist ungerade, zahlt A Taler an B.

Sollte man bei dem Spiel

(1) lieber A sein?
(2) lieber B sein?
(3) oder ist das egal?
xb Auf diesen Beitrag antworten »

Lösung 64

(Das müsste so reichen.Es gibt da noch die Hesse Matrix.Aber braucht man hier glaube ich nicht)

Zunächst die Wahrscheinlichkeiten





Dann der Gewinn von A














in die Gewinnfunktion einsetzen



Das Spiel ist aus Sicht von B besser

HAL 9000 Auf diesen Beitrag antworten »

Wer vielleicht Zweifel hat, wieso der so ermittelte Sattelpunkt der Funktion den Spielwert repräsentiert, kann hier einen Crashkurs in Sachen Zweipersonen-Nullsummenspiel belegen. Augenzwinkern
xb Auf diesen Beitrag antworten »

Aufgabe 65

Es wurden zeitgleich 2 Fotos von der Sonne gemacht
Ein Fotograph befand sich in Basel (Schweiz) und einer in Sao Paulo (Brasilien)

Um wieviel Grad ist die Sonne auf den beiden Bilder gedreht?

Die Aufnahmen wurden am 14.7.2017 gemacht.
In der Schweiz war es 20Uhr (Sommerzeit)

Die Kameras waren natürlich horizontal ausgerichtet
(Dutch Angle=0°)


Diese Seite könnte hilfreich sein
https://www.timeanddate.de/astronomie/ta...o=20170621T1330[/quote]
HAL 9000 Auf diesen Beitrag antworten »

Geklaut von Dopap:

Zitat:
Aufgabe 66

Aus einer Urne mit weißen sowie schwarzen Kugeln werden alle Kugeln gezogen unter Beachtung der Ziehungsreihenfolge. Die Zufallsgröße bezeichne die Anzahl der runs innerhalb des Ziehungsergebnis, dabei bezeichne ein run eine (maximal ausgedehnte) ununterbrochene Folge gleichfarbiger Kugeln. Man zeige die Formeln für

a) Wahrscheinlichkeitsverteilung

,

Gültig für alle natürlichen ; für alle ergibt sich aber aufgrund der Binomialkoeffizienten automatisch Wert Null.

b) Erwartungswert .

c) Varianz .

Hinweis: Man kann b),c) auf Grundlage von a) berechnen, muss es aber nicht. Man kann auch auf Basis von argumentieren, wobei die Indikatorvariable genau dann gleich 1 ist, wenn Kugel und verschiedenfarbig sind.
klauss Auf diesen Beitrag antworten »

Da ich gerade Zeit und Lust zu einer kleinen Knobelaufgabe hatte, habe ich mir diese wegen ihres überschaubaren Aufwands herausgesucht und stelle dazu eine Alternative zur Verfügung.

Zitat:

Lösung Aufgabe 60

Ausgehend von

forme ich weiter um zu

Für die ersten paar Quadratzahlen lassen sich schnell die geeigneten Wertepaare von und finden, mit deren Gesetzmäßigkeit alle Lösungen auch rekursiv angeben werden können.
Seien die möglichen Wertepaare von und , geordnet durch , bei vertauschbarer Zuordnung von und .
Dann gilt für :



Leopold Auf diesen Beitrag antworten »

Angeregt durch Huggys Beitrag hier eine kleine Aufgabe zum Jahresschluß.

Zitat:
Aufgabe 67

seien vier Punkte des dreidimensionalen Anschauungsraumes, so daß keine drei kollinear sind. Im geschlossenen Streckenzug mögen aufeinander stoßende Kanten jeweils einen rechten Winkel bilden.

Man beweise oder widerlege:
ist ein Rechteck.
klauss Auf diesen Beitrag antworten »

Ich versuchs mal wieder vektoriell.

Zitat:

Lösung Aufgabe 67

Geg.:





Geschlossener Vektorzug:


Gleichung 4 mal mit Skalarprodukt durchmultipliziert:







Gegenüberliegende Seiten sind also gleich lang.

Damit kann ich oben einsetzen:




Analog:

Gegenüberliegende Seiten sind parallel.

Es handelt sich um ein Rechteck.



Bitte um Kontrolle.
Leider habe ich keine geeignete Folgeaufgabe zur Hand.
Leopold Auf diesen Beitrag antworten »

Deine Lösung scheint mir richtig. Ich habe es ebenfalls mit Analytischer Geometrie gemacht. Mir schien es aber einfacher, Koordinaten zu Hilfe zu nehmen.

Man kann die Figur so in ein kartesisches -Koordinatensystem legen, daß der Ursprung wird und auf der -Achse beziehungsweise -Achse liegen. Mit kann man daher



ansetzen. Aus den Orthogonalitätsbedingungen folgt mit Hilfe des Skalarprodukts:



Die Punkte mit ihren Koordinaten eingesetzt, erhält man aus den ersten beiden Bedingungen , also und , also . Somit ist . Das verwendet man gleich in der dritten Bedingung oben und erhält , also .

Damit ist und ein Rechteck.

Es gibt also keinen geschlossenen Vierstreckenzug mit lauter rechten Winkeln, der kein Rechteck ist.
HAL 9000 Auf diesen Beitrag antworten »

Zitat:
Aufgabe 68

Es sei . Die Zahlenmenge wird disjunkt in zwei Teilmengen A und B aufgeteilt, d.h., jede dieser ganzen Zahlen landet dabei in genau einer dieser beiden Teilmengen.

Man zeige, dass man in mindestens einer der beiden Teilmengen zwei verschiedene Zahlen findet, deren Summe eine Quadratzahl ist.

Stammt aus einem abgeschlossenen Wettbewerb. Augenzwinkern
HAL 9000 Auf diesen Beitrag antworten »

Noch eine weitere Aufgabe aus diesem Wettbewerb:

Zitat:
Aufgabe 69

Die beiden Eichhörnchen Buschi und Hoppi haben 2021 Walnüsse für den Winter gesammelt. Hoppi nummeriert die Walnüsse von 1 bis 2021 und gräbt 2021 kleine Löcher in den Boden entlang eines Kreises um ihren Lieblingsbaum. Am nächsten Morgen bemerkt Hoppi, dass Buschi in jedes Loch eine Walnuss gelegt hat, aber ohne die Nummerierung zu beachten. Verstimmt beschließt Hoppi, die Walnüsse umzuordnen, und führt dazu eine Folge von 2021 Zügen aus. Im -ten Zug vertauscht Hoppi die Positionen der beiden Walnüsse, die direkt neben Walnuss liegen. Man beweise, dass es eine ganze Zahl derart gibt, dass Hoppi im -ten Zug zwei Walnüsse und mit vertauscht.
Huggy Auf diesen Beitrag antworten »

Zitat:
Lösung Aufgabe 68

Eine Aufteilung des Intervalls in 2 Teilmengen, bei der in mindestens einer der beiden Teilmengen die Summe zweier Zahlen aus dieser Teilmengen eine Quadratzahl ist, nenne ich eine Q-Aufteilung. Es ist dann zu beweisen, dass für jede Aufteilung eine Q-Aufteilung ist. Man betrachte nun die Lösungen des Gleichungssystems















Wenn alles natürliche Zahlen sind und , dann gibt es bei jeder Aufteilung der Zahlen in in 2 Teilmengen eine Teilmenge, in der die Summe zweier Elemente eine Quadratzahl ist, weil ja von den 3 Zahlen immer mindestens 2 in einer der beiden Teilmengen liegen. Damit natürliche Zahlen sind, muss ungerade sein. Man kann zunächst auf einschränken. Da sich in der Folge aber eine stärkere Einschränkung ergeben wird, lasse ich die Details dazu weg. Damit der nächste und entscheidende Schritt anschaulicher wird, bringe ich zunächst ein paar Beispiele.


ergibt . Man kann schließen, dass in dem Intervall jede Aufteilung eine Q-Aufteilung ist.


ergibt . Man kann schließen, dass in allen Intervallen mit jede Aufteilung eine Q-Aufteilung ist. Für hat man so keinen Beweis, dass jede Aufteilung eine Q-Aufteilung ist.


ergibt . Man kann schließen, dass in allen Intervallen mit jede Aufteilung eine Q-Aufteilung ist. Für hat man so keinen Beweis, dass jede Aufteilung eine Q-Aufteilung ist.

Wenn es zwischen und keine Lücke gibt, gilt der Beweis für alle . Das führt zu der Gleichung



mit der Lösung , d.h.
HAL 9000 Auf diesen Beitrag antworten »

Ja, die Idee klappt für alle und auch noch für ein paar weitere kleinere . Freude

Ich hab mal rumprobiert: Für gibt es wohl tatsächlich eine Aufteilung von , für das man kein solches Quadratzahlsummen-Paar findet.

A : 98-126 , 129-161 , 162-196
B: 99-127 , 128-160 , 163-195

(die Bereiche sind immer in Zweierschritten zu verstehen, d.h., 98-126={98,100,...,126} usw.). Den insgesamt nur in Frage kommenden Quadratsummen 225, 256, 289, 324 und 361 geht man damit erfolgreich aus dem Weg.
HAL 9000 Auf diesen Beitrag antworten »

Und noch eine (insgesamt wohl die kniffligste von den dreien):

Zitat:
Aufgabe 70

Es sei eine konkave Funktion mit Minimum im linken Intervallrandpunkt 0. Man zeige, dass für alle reellen Zahlen die Ungleichung



gilt.

Im Original war die Behauptung "nur" für zu zeigen.
HAL 9000 Auf diesen Beitrag antworten »

Es gibt eine verblüffend einfache Lösung zu Aufgabe 69, passend für beliebige ungerade Walnussanzahlen :

Wir markieren nach dem -ten Schritt die Walnuss Nr. und bestimmen anschließend die Anzahl von Nachbarpaaren, wo BEIDE Nüsse markiert sind.

Angenommen, die Behauptung ist falsch, dann ist unmittelbar VOR dem -ten Schritt die Nuss unmarkiert, und ihre beiden Nachbarn entweder beide unmarkiert (das ist der Fall ) oder aber beide markiert (das ist ). In beiden Fällen ändert der Tausch der beiden Nüsse nichts an der Anzahl der markierten Paare. Durch das nun erfolgte Markieren der Nuss bekommen wir im ersten Fall , und im zweiten Fall , d.h., die Parität von bleibt stets erhalten.

Start ist (gerade) vor dem ersten Schritt - klar, denn da sind ja noch gar keine Nüsse markiert, erst recht keine Nachbarpaare.

Schluss ist , denn da alle Nüsse dann markiert sind, trifft dies auch auf alle möglichen Nachbarpaare in dieser Kreisanordnung der Nüsse zu. Ist nun aber ungerade, dann widerspricht das der Paritätserhaltung für mindestens ein . Damit ist in diesem Fall die Annahme falsch, und somit die Behauptung richtig.
HAL 9000 Auf diesen Beitrag antworten »

Im diesem Thread stand letztens ein Beitrag, der überwiegend aus Spam und Zahlenmystik bestand und deshalb wohl weggeräumt wurde, aber immerhin folgende interessante Aussage enthielt:

Zitat:
Aufgabe 71

Die Folge sei definiert durch und für Man zeige, dass für jede ungerade Primzahl die Zahl durch teilbar ist.
HAL 9000 Auf diesen Beitrag antworten »

Ich hab irgendwie das Déjà-vu, als hätte ich das schon mal im Board gepostet, finde es aber nicht mehr - daher bitte ich schon mal prophylaktisch um Verzeihung, falls dem wirklich so ist:

Zitat:
Aufgabe 72

Es sei eine divergente Reihe mit positiven Gliedern. Man zeige, dass es dann eine ebenso divergente Reihe mit positiven Gliedern gibt, die "langsamer" als die erste Reihe divergiert, d.h., für die gilt.



Inspiriert ist diese Problemstellung durch folgende Kette divergenter Reihen:



(die Anfangsindizes sind jeweils so groß zu wählen, dass die Terme alle definiert und positiv sind).
Huggy Auf diesen Beitrag antworten »

Man kann das aus dem Satz von Abel-Dini

https://de.wikipedia.org/wiki/Sapogowsches_Kriterium

folgern. In dem Link ist



die divergente Folge mit positiven Gliedern und



sind ihre Partialsummen. Setzt man



so ist nach dem Satz auch



divergent und es ist



und das geht gegen Null, weil ja nach Vorraussetzung divergiert.
HAL 9000 Auf diesen Beitrag antworten »

Ja, genau diese Konstruktion hatte ich auch im Sinn, und dafür dann einen elementaren Beweis. Vielen Dank für diesen Verweis auf "Sapogowsches Kriterium", kannte ich bisher noch nicht. Freude
Huggy Auf diesen Beitrag antworten »

Dieses Kriterium ist bei der Suche nach einem Link für den Satz von Abel-Dini zufällig aufgetaucht. Ich kannte es bisher auch nicht.

Ein Versuch zu einem elementaren Beweis des Satzes von Abel-Dini im Falle der Divergenz: Man hat



Da divergiert, kann man zu jedem ein wählen, so dass



Dann hat man



Wäre konvergent, müsste es aber zu jedem ein geben, so dass für alle die vorige Summe ist. Das führt bei zu einem Widerspruch.
HAL 9000 Auf diesen Beitrag antworten »

Ich klau mal von https://www.onlinemathe.de/forum/Stetigkeit-ffx-x eine Aufgabe, die dort mangels Interesse des Fragestellers nicht mehr wirklich besprochen wurde:

Zitat:
Aufgabe 73

Es sei eine Funktion, für die für alle gilt.

a) Man zeige, dass nicht stetig ist.

b) Man gebe eine Lösungsfunktion an.
IfindU Auf diesen Beitrag antworten »

Aufgabe 73.

a) Aus folgt und damit bijektiv. Wenn stetig und bijektiv ist, ist es strikt monton. Mit monoton, folgt ist monoton wachsend. Jetzt ist aber strikt monoton fallend. D.h. ist unstetig.

b) Wir partitionieren mit und . Definiere

.
D.h. die idee ist, dass anwenden von die Werte in die Mengen schickt: .

Ziemlich cool, wo eine schöne komplexe Funktion ist mit .

Edit: Ich glaube das ist falsch/unsauber. Die Mengen und bestehen aus links-offenen-rechts-abgeschlossenen Intervallen. Diese müssten anders herum...
Neue Frage »
Antworten »



Verwandte Themen

Die Beliebtesten »
Die Größten »
Die Neuesten »